TBR: Weird Experiment with Lenz's Law: Explain Please?!

This forum made possible through the generous support of SDN members, donors, and sponsors. Thank you.

justadream

Full Member
10+ Year Member
Joined
Apr 29, 2011
Messages
2,171
Reaction score
862
TBR Physics Book II page 153 #10

f2.jpg


Before I even ask about the TBR question, can someone explain how Experiment II works? I don’t understand how you can suspend the bar by having a current between points A and F.


Here’s the TBR Question “In Experiment II, starting when the bar is suspended, what will be observed when the switch between points A and F is opened?

Answer: The current through the bar goes to zero, and the bar descends to the bottom of the rails”

TBR’s explanation: “In Experiment II, before the switch between A and F is opened, there is current flowing between points A and F that is in opposite direction of the current flowing between points X and Y [WHAT? I don’t understand this at all] . Once the switch is opened, the current stops and there is no longer a force suspending the bar. Without any force holding the bar up, it will slide down the rails due to gravity, eventually reaching the bottom.”

Members don't see this ad.
 
Experiment 2 creates a current that goes from a > x > y > f - imagine this as a current carrying loop.

This loop will have a sort-of counterclockwise current that will generate a upwards pointing magnetic field (in relation to the tangent of the rails).

The field will point away from the center of the loop in essence.

F (on bar) = IL x B which will generate a force to the right. This force will push the bar to the right and up the rails.
 
@Cawolf

So if you have a CCW current, then yes, I can see how the magnetic field inside the loop will point outwards (out of the loop).

But how do you get it to generate a force to the right?

If I'm using the right-hand rule:

Thumb = Velocity (***idk what it would be here***)
Fingers = Field (pointing outward)
Palm = Force
 
You would use F = IL x B

I use A x B = C (A = index finger, b = middle finger, c = product)

I = along bar from x to y
B = out of loop
F = to the right up the bars

The F = qv x B is for a moving charged particle, we want to use the formula for current through a wire (the bar).
 
Members don't see this ad :)
@Cawolf

I've never seen that way to do it? Can't you just have the "I" take the place of the "v" in the F=qvb right-hand-rule?

In any case, don't you also need to give it an initial push?
 
Yes essentially. If you want to think of it that way, the IL term will replace the qv term. Those both have the same units of [A][m].

Here you do not, the little box is said to generate a potential that results in current flow through the bar - generating the B field.
 
@Cawolf

So even though the bar is initially not moving, inducing a current can get it moving?

Also, how do you know which direction the current is induced from the box?
 
Yes. Turning on the battery generates a current, which generates the force.

It doesn't matter which way the current flows. If you check your right hand rule you will see that a CCW current generates a upward B field - exerting a rightward force on the bar.

If a CW current is generated, then the B field is down and the force is still to the right.

There is no external field in this experiment, moving the bar does not generate a current.
 
@Cawolf

So why does TBR say "before the switch between A and F is opened, there is current flowing between points A and F that is in opposite direction of the current flowing between points X and Y"

Shouldn't it be CCW or CW at both locations?
 
Experiment 2 creates a current that goes from a > x > y > f - imagine this as a current carrying loop.

That's why I said that. I was just mentioning that it really doesn't matter with respect to the force on the bar.
 
@Cawolf

I see now that the direction of the current in the loop doesn't matter with respect to the force but between points AF (that is, point A to F) and XY, the current should still be moving in the same direction right (as in, moving CW in both places or moving CCW in both places)?

Also, to compare it with my other current loop question (the one with an external magnetic field - here there is only an induced one):

In the other example (external magnetic field), the bar cannot move unless it was already moving. The current generated in that loop is induced.

Here in this example, there is an applied current through the loop (which generates a magnetic field). Is it this induced magnetic field that is causing the bar to feel a force? But I thought magnetic force = 0 if the bar were initially not moving. I'm still unclear about how it starts initially moving.
 
The current is moving CCW in both places but they are just saying it goes in opposite directions because they are opposite sides of a loop.

Yes - the applied current generates the field that causes the force on the bar.

The bar is not moving, but charge is moving through it.

I = dQ/dt

That is why you should model this bar as a wire and use the formula that F = IL x B.
 
@Cawolf

Okay so before the bar move, I think the thing that is "moving" are the CHARGES in the current?

That is what causes the force.
 
Yes - current is moving charge.The moving current experiences the force - and the current is in the bar.
 
  • Like
Reactions: 1 user
@Cawolf

Okay I think I understand how it moves up now.

So while it is moving up (increasing the area of the current loop), does Lenz's law act in some way to alter its motion?
 
No Lenz's law applies to an external field. This loop is generating the field - there is no external field.

So there is no change in flux. It does not apply.
 
@Cawolf

So the fact that that the loop is increasing in area doesn't matter for Lenz's Law?

Would the increase in area at least increase the force (F = ILB) and L is getting larger?
 
L is the length of the bar - that is constant.

Lenz's Law only applies to some external field and it's flux through the loop - there is no external field.
 
@Cawolf

Isn't the current going around in a loop?

As the bar moves to the right, the current loop length increases right?

I guess you're saying that since the only part that is "moveable" is the bar, we should only consider the length of the bar?

In that case, then the other non-moveable parts of the apparatus (in which current is flowing - such as from A to X) feel a force but don't move because they aren't moveable?
 
Isn't the current going around in a loop?
Yes it is.

As the bar moves to the right, the current loop length increases right?

Yes - but L is defined as the length of the bar which feels the force.

I guess you're saying that since the only part that is "moveable" is the bar, we should only consider the length of the bar?
Yes.

In that case, then the other non-moveable parts of the apparatus (in which current is flowing - such as from A to X) feel a force but don't move because they aren't moveable?

Yes. Generally you would only have a torque on the loop, but here the movable part experiences a force from the other parts of the apparatus.
 
@Cawolf
Okay thanks for that! Going back to the Lenz's law:

This may sound stupid:

You said Lenzs law only applies when there is an external magnetic field.

In this case, there isnt an external mag field. Instead, an induced field is made.

Okay so lets say at moment X there is an induced field made. At moment X+.0000001 seconds why wouldn't the induced field at moment X act as an "external field" that could then have Lenz-law implications at moment X+.0000001 seconds?
 
We are getting to a point that I don't feel I can speak about with high confidence. I am not a physicist. I just apply the concepts I have learned to problems.

Lenz's Law deals will flux. There is no net flux in this situation. The field goes out and comes back in. There are no magnetic dipoles. Even though we consider the field to be straight through the loop - the magnetic field eventually loops around back to where it started. An external field can pass through the loop in one direction - creating flux.

This picture is not great, but you can see the curved path of the B field.

curloo.gif
 
  • Like
Reactions: 1 user
Top